Série de Fourier douteuse ?

Mister Da
Modifié (January 2022) dans Analyse
Bonjour,
sur internet, je suis tombé sur tout un tas de discussions autour de séries de Fourier de fonctions du type : $x\mapsto \ln(1-\cos(x))\ ``=" -\ln(2) -2\sum_{n=1}^{+\infty}\frac{\cos(nx)}{n}$. Je ne sais pas dans quel sens il faut accepter cette égalité, déjà on est mal parti car il y a des problèmes pour $x\equiv 0\mod 2\pi$, la fonction semble fâchée avec Abel et Dirichlet et sans qui je suis perdu.

Certains font de sauvages calculs des coefficients $a_n$ de Fourier mais il y a toujours des choses étranges dans le calcul des intégrales à base de $0\times \infty = 0$ sans de justifications.

D'autres utilisent la formule $\ln(1- z) = -\sum_{n=1}^{+\infty}\frac{z^n}{n}$ avec $z=\mathrm{e}^{ix}$ en occultant le fait qu'en toute rigueur la convergence est seulement pour $|z|<1$.

Y aurait-il un moyen (pas trop technique) de prouver qu'il y a convergence sur la frontière du disque de convergence dans ce cas précis ? Naïvement, je pensais faire le calcul avec $\ln(1- r\mathrm{e}^{ix})$ et $r<1$ mais je ne suis pas en mesure d'inverser la somme et la limite pour faire tendre $r$ vers $1$ ensuite.

Numériquement (bien que ça ne soit pas une preuve) je trouve le résultat bluffant, sans avoir à pousser la somme trop loin (ici je pousse $n$ à $60$) la série de Fourier (rouge) accroche bien la fonction (bleu).
Cordialement,
Mister Da

Réponses

  • C'est une application classique du théorème d'Abel : la série $\sum_n \frac{z^n}{n}$ converge pour tout $z \neq 1$ de module $1$, et le théorème nous dit en particulier que la fonction somme (définie a priori sur le disque ouvert de convergence) admet pour limite en $z$, la somme $\sum_{n=0}^{+\infty} \frac{z^n}{n}$.
    Par continuité de la fonction $z \mapsto \log(1-z)$ sur par exemple $\{z \in \mathbb C \mid |z| < 1+\varepsilon, z \not \in [1, 1+\varepsilon[\}$, on en déduit que ladite somme est bien $-\log(1-z)$.

  • Bonjour
    Ce qui est certain  c'est que l'égalité  a lieu dans $L^2(0,2\pi).$  Si tu démontres que la série de Fourier est continue sur $]0,2\pi[$  alors l'égalité est vérifiée pour tout $x\in]0,2\pi[$.  
     
  • Mister Da
    Modifié (January 2022)
    Bonjour,
    @Poirot. Merci ! C'est Abel que j'avais mal digéré, les sommes partielles de la suite $\left((\mathrm{e}^{\mathrm{i}\alpha})^n\right)_n$ sont bien bornées si $\alpha\not\equiv 0\mod 2\pi$, c'est ici que je suis allé trop vite (alors qu'il s'agit suite suite géométrique).
    @bd2017. Merci. Si j'arrive à démontrer la continuité sur $]0,2\pi[$, pourrais-je conclure à la converge uniformément de la série sur tout compact inclus dans $]0,2\pi[$ ? Je pense que oui mais j'ai l'impression de m'enflammer un peu trop vite (j'ai déjà tellement fait mentir Fourier que je n'ose plus jamais rien dire).
    Cordialement,
    Mister Da
  • gebrane
    Modifié (January 2022)
    Mister Da dit Naïvement, je pensais faire le calcul avec ln(1−reix)l et r<1 mais je ne suis pas en mesure d'inverser la somme et la limite pour faire tendre r vers 1 ensuite

    Tu peux utiliser ce théorème (ce qu'il me reste en mémoire) bien connu en analyse de Fourier
    Theorem:  Soit $f(\theta)= \sum_{n\in\mathbb{Z}}c_n e^{in\theta}$. Si $f\in L_2(\mathbb{S}^1)$ , alors   $\lim_{r\to 1}\sum _{n\in\mathbb{Z}}r^{|n|}c_ne^{in\theta}=\sum_{n\in\mathbb{Z}}c_n e^{in\theta}$ dans  $L_2$ et en tout point de   Lebesgue de $f$

    Ce théorème te permet de passer à la limite quand $r\to 1$ dans $ -\log(1-re^{i\theta})=\sum_{n\geq1}\frac{r^ne^{ni\theta}}{n}$
    Le 😄 Farceur


  • YvesM
    Modifié (January 2022)
    Bonjour,


    Erreur de calcul… j’ai calculé $\ln \cos x$…
    $\displaystyle 1-\cos x = {e^{i x} \over 2 } (1+e^{-2 i x})$ pour tout $x$ réel.

    Donc, pour tout $x$ réel tel que $\displaystyle 1-\cos x \neq 0$, $\displaystyle \ln(1-\cos x) = - \ln 2 + i x + \ln (1+e^{-2 i x}).$
    Et donc, pour tout $x$ réel, tel que $\displaystyle 1-\cos x \neq 0$ et tel que $\displaystyle |e^{-i 2 x}|<1$, on a $\displaystyle \ln (1-\cos x) = - \ln 2 + i x - \sum_{n \geq 1} {e^{-2i n x} \over n}.$
    On prend alors les parties réelle et imaginaires. 
    La formule du message original est fausse (typo sur un facteur $2$). 

    On peut choisir $x \in ]0,\pi[.$
  • Mister Da
    Modifié (January 2022)
    Bonjour,
    @gebrane, merci, je ne connaissais pas ce théorème.
    @YvesM, je n'ai pas compris l'erreur de typo que j'ai faite.
    En repartant de ce que tu proposes, on a, $\ln(1-\cos x) = - \ln 2 + i x + \ln (1+e^{-2 i x})$ mais aussi $\ln(1-\cos x) = - \ln 2 - i x + \ln (1+e^{+2 i x})$. En faisant la moyenne des deux et qu'on continue ce que tu fais on retombe bien sur ce que j'ai écrit non ?
    Cordialement,
    Mister Da.

  • YvesM
    Modifié (January 2022)
    Bonjour
    Erreur de mon côté… 
    Non. Tu fais une erreur de calcul.
    La somme divisée par deux de $e^{2 i\pi x n}$ et $e^{-2 i \pi x n}$ est $\cos(2 n x)$ et non pas $\cos(n x)/2.$ 
  • YvesM a dit :
    $\displaystyle 1-\cos x = {e^{i x} \over 2 } (1+e^{-2 i x})$ pour tout $x$ réel.

    L'erreur serait plutôt ici non ?
    Cordialement,
    Mister Da
  • jandri
    Modifié (January 2022)
    On peut démontrer de façon élémentaire (sans rien connaitre des séries de Fourier ni des fonctions de variable complexe) que pour $x\in]0,2\pi[$ : 
    $$\sum_{n=1}^{+\infty}\frac1n\cos(nx)=-\frac12\ln(2-2\cos x)=-\ln(2\sin(x/2)).$$
    On obtient même la convergence uniforme sur tout segment inclus dans $]0,2\pi[$.
    Pour cela il suffit d'écrire $\displaystyle\sum_{n=1}^N\dfrac1n\cos(nx)=\Re \displaystyle\int_0^1\sum_{n=1}^Nt^{n-1}e^{inx}dt=\Re \displaystyle\int_0^1\dfrac{e^{ix}}{1-te^{ix}}dt-R_n(x)$ avec $|R_n(x)|\leqslant\displaystyle\int_0^1\dfrac{t^N}{|1-te^{ix}|}dt\leqslant\dfrac{1}{m(N+1)}$ si $x\in[a,b]\subset\,]0,2\pi[$.
    Ensuite $\Re \displaystyle\int_0^1\dfrac{e^{ix}}{1-te^{ix}}dt=\displaystyle\int_0^1\dfrac{\cos(x)-t}{1-2t\cos(x)+t^2}dt=-\frac12\ln(2-2\cos x)=-\ln\big(2\sin(x/2)\big)$.
  • bd2017
    Modifié (January 2022)
    @Mister Da  En réponse à ta question la réponse est oui.  Mais je ne sais pas si le résultat est standard. En tout cas la preuve n'est pas triviale.
    D'autre part   pour avoir calculé la série de Fourier  je ne vois pas d'erreur dans la formule initiale.
    Et puis  $1 -\cos(x)= \exp(ix)  /2  (1+\exp(-2 ix)  $ c'est manifestement faux.
     
  • Bonjour,
    merci pour vos réponses.
    @bd2017 j'ai refait également mes calculs et j'ai vérifié ce que j'avais programmé pour obtenir le graphe et tout est cohérent avec la formule initiale. Concernant la convergence uniforme jandri a posté un message juste avant toi.
    @jandri, merci, je vais refaire tes calculs calmement pour voir si je comprends tout mais étant d'une lenteur exceptionnelle il se peut que je mette du temps.
    Cordialement,
    Mister Da
  • Ok  je n'avais pas vu le message de  @Jandri.  C'est exactement ce qu'il  faut faire.  La fonction n'étant pas bornée, on est à la limite des résultats usuels et il faut faire le travail  à la main comme cela .
     
  • Bonjour,
    j'ai pu enfin me pencher à nouveau sur la question. Merci @Jandri pour ton calcul et les détails tout est super limpide. Ton calcul est super astucieux et j'aurai été bien incapable de le trouver !
    Merci à tous pour votre aide, j'ai appris énormément de choses dans cette discussion et je vous en suis très reconnaissant.
    Cordialement,
    Mister Da
  • L'astuce principale (qui n'est pas de moi) est d'écrire $\dfrac1n=\displaystyle\int_0^1t^{n-1}dt$ pour faire apparaitre une somme partielle de série géométrique (en écrivant de plus $\cos x=\Re (e^{ix})$ mais c'est très classique).
  • bd2017
    Modifié (January 2022)
    Bonjour
    Une remarque quant à la convergence uniforme sur $[\epsilon, 2\pi -\epsilon].$   
    Il y a un résultat qui dit que si  les  coefficients  $a_k$  et  $b_k $   d'une  série trigonométrique  $\sum a_k \cos (  kx) + \sum b_k \sin (k x)$ sont  à
    valeurs  dans $\R^+$   et s'ils sont décroissants alors la série CVU  sur $[\epsilon, 2\pi -\epsilon]$  (avec $\epsilon\in\,]  0 ,\pi[$ ). 
    Il est bien de savoir que cela se démontre avec l'aide d'une transformation d'Abel  et le critère de Cauchy uniforme. 
    Si ce résultat est connu, on peut alors  l'appliquer et ceci  donne une alternative à la démonstration de @Jandri.  
     
  • Mister Da
    Modifié (January 2022)
    @jandri c'est surtout à la première astuce à laquelle je n'aurais pas pensé l'ombre d'une seconde ! Ca fait longtemps que je n'ai plus pratiqué et je constate que je rouille à vue d'œil.
    @bd2017 Pour m'amuser je voulais calculer explicitement les coefficients $a_k$ mais pour le moment je patine. Je vais essayer du coup d'étudier ce que tu viens de me souffler. Pour simplifier on se concentre sur les $a_k$. On les considère positifs décroissants et bon j'imagine tendant vers $0$.
    Convergence ponctuelle
    Pour $x\not\equiv 0\mod 2\pi$ fixé, on considère les sommes partielles $S_N(x) = \sum_{k=1}^N a_k\cos (kx)$ et $T_N(x) =\sum_{k=1}^N \cos (kx)$.
    Avec la reformulation d'Abel on a $S_N(x) = a_NT_N(x) - \sum_{k=1}^{N-1} (a_{k+1} - a_k)T_k(x)$.
    Comme $T_N(x)= \frac{\sin(\frac{N}{2}x)}{\sin(\frac{x}{2})}\cos(\frac{N+1}{2}x)$ (aux erreurs de calcul près), on constate que la suite est bornée et on note $M(x)$ un majorant des $|T_k(x)|$. Ainsi, $|a_NT_N(x)|$ tend vers $0$ car $a_N$ tend vers $0$ et $T_N(x) $ est borné et la suite est absolument convergente donc convergente, en effet, la suite étant décroissante : $|a_k - a_{k+1}| = a_k - a_{k+1}$ et  $M(x)\sum_{k=1}^{N-1} (a_k - a_{k+1}) = M(x)(a_1-a_N)$ tend vers $M(x)a_1$.
    Convergence uniforme
    On se donne un intervalle $I=[\varepsilon, 2\pi-\varepsilon]$ avec $\varepsilon\in]0,\pi[$. La convergence uniforme est : pour tout $\alpha>0$, il existe un entier $N_\alpha$ tel que pour tout $p,n>N_\alpha$ on ait, pour tout $x\in I$, $|S_p(x) - S_n(x)|<\alpha$.
    Pour tout $p>n$, on note \[S_{n,p}(x) = S_p(x) - S_n(x) =\sum_{k=n+1}^p a_k\cos (kx)\quad\mathrm{et}\quad T_{n,p}(x) =\sum_{k=n+1}^p \cos (kx) = \cos((p+n+1)x/2)\frac{\sin((p-n)x/2)}{\sin(x/2)}\;.\]Par Abel, \[S_{n,p}(x) =  a_pT_{n,p}(x) + \sum_{k=n+1}^{p-1} (a_k- a_{k+1})  T_{n,k}(x)\;. \]Pour tout $x\in I$, et pour tout $p$ et $n$, $T_{n,p}(x) \leq \frac{1}{\sin(\varepsilon/2)}$, ainsi \[|S_{n,p}(x)| \leq  \frac{1}{\sin(\varepsilon/2)}\left[ a_p + \sum_{k=n+1}^{p-1} (a_k- a_{k+1})\right] =  \frac{a_{n+1}}{\sin(\varepsilon/2)}\;.\]
    Il suffit de prendre un $N_\alpha$ tel que $a_{N_\alpha} < \sin(\varepsilon/2)\alpha$. Comme la suite $(a_k)_k$ est décroissante et tend vers $0$, ce $N_\alpha$ existe.
    J'ai galéré comme pas possible, c'est sans doute plein de fautes mais bon, je ne suis pas allé voir sur le net.
    Cordialement,
    Mister Da

    Édit : correction mineure
  • bd2017
    Modifié (January 2022)
    Pour  calculer les coefficients de Fourier de $f(x)=\ln(1 - \cos(x))$  on peut par exemple faire une intégration par parties 
    pour être ramené au calcul de $I_n=\int_0^{2 \pi}\frac{\sin(x) \sin (n x)}{1-\cos(x) } dx $
    Pour faire ce calcul  on peut passer aux complexes  et faire le changement de variable $z=exp(i x) $   et appliquer le th des résidus. 
    Il y   a surement d'autres moyens  comme par exemple développer  $\ln(1 - \cos(x))$  en série .     Mais je n'ai pas regardé.
     
  • Mister Da
    Modifié (January 2022)
    Bonjour,
    oui, j'ai tenté avec $\ln(2\sin(x/2)^2)$ et j'ai également tenté une intégration par parties j'obtiens bien ton $I_n$ sur lequel je bloquais, je n'avais pas pensé aux résidus, je faisais des formules trigonométriques dans tous les sens désespérément. Mais je suis honteux, comment on montre que le crochet $\frac{1}{n}\left[\sin(nx)\ln(1-\cos(x))\right]_0^{2\pi}$ est nul ?
    Cordialement,
    Mister Da
  • gerard0
    Modifié (January 2022)
    Bonjour.
    Comme il s'agit de limites, des équivalents et le fait que x*ln(x) tend vers 0.
    Cordialement.
  • Mister Da
    Modifié (January 2022)
    Bonjour,
    merci gerard0 ! Entre temps, je pense avoir fait une chose équivalente (sans mauvais jeu de mots) en appliquant le théorème de l'Hospital à la quantité $\dfrac{\ln(1-\cos(x))}{\frac{1}{\sin(nx)}}$. Je ne sais pas pourquoi mais hier je n'y ai pas pensé.
    Cordialement,
    Mister Da
  • Re bonjour,
    @bd2017, désolé de te solliciter à nouveau. Je viens seulement de me pencher sur le calcul de $I_n$ via les résidus histoire de me coucher moins ignorant ce soir. C'est assez laborieux, sommes-nous d'accord pour dire que le calcul n'est pas si direct que cela étant donné que la singularité est sur le bord du disque centrée de rayon $1$ ou ai-je encore loupé un truc ?
    D'autre part, je viens de relire ce que j'ai écris sur la convergence uniforme, je n'ai pas vu de trou dans la raquette. Etait-ce ce genre de démonstration que tu avais en tête ?
    Merci par avance.
    Cordialement,
    Mister Da
  • Re bonjour,
    @bd2017, désolé de te solliciter à nouveau. Je viens seulement de me pencher sur le calcul de $I_n$ via les résidus histoire de me coucher moins ignorant ce soir. C'est assez laborieux, sommes-nous d'accord pour dire que le calcul n'est pas si direct que cela étant donné que la singularité est sur le bord du disque centrée de rayon $1$ ou ai-je encore loupé un truc ?
    D'autre part, je viens de relire ce que j'ai écris sur la convergence uniforme, je n'ai pas vu de trou dans la raquette. Etait-ce ce genre de démonstration que tu avais en tête ?
    Merci par avance.
    Cordialement,
    Mister Da
  • bd2017
    Modifié (January 2022)
    Bonjour     1 .Pour  la CVU oui  c'est ça.  Disons que la démonstration de @Jandri l'emporte ici.  Mais la démonstration que tu fais peut servir dans le cas où les $a_n$ ne facilite pas les calculs  mais fonctionne si on sait  que la suite tends vers  0  en décroissant.
    2. Pour le calcul de $I_n,$  $1$   n'est pas une singularité puisque  tu  dois avoir un facteur de la forme  $(z^n-1)/(z-1) =\sum_{p=0}^{n-1}  z^p.$ Ainsi la seule singularité est $z=0$.
     
  • Mister Da
    Modifié (January 2022)
    Bonjour
    merci énormément. Effectivement, l'approche de Jandri est le truc efficace à faire mais comme j'étais venu pour visiter la région j'avais envie de faire la démonstration de ta solution alternative.
    Pour $I_n$, j'ai fait des erreurs de calculs et de recopie bref j'étais mal parti. Merci pour ton indice. Du coup je suis parti de
    \[f(z) = \frac{1}{iz} \frac{1}{-4} \frac{ (z-z^{-1})(z^n-z^{-n})}{1-\frac{(z+z^{-1})}{2}}\]
    et en débroussaillant (correctement) on arrive à \[f(z) = \frac{1}{2i}(z^n+1)(z+1)\frac{1}{z^{n+1}}\frac{z^n-1}{z-1}\]
    et la seule singularité est donc en $z=0$. Tu viens de me faire gagner une soirée !
    Cordialement,
    Mister Da
Connectez-vous ou Inscrivez-vous pour répondre.